You are on page 1of 36

LEGALEDGE TEST SERIES

Part of the Most Comprehensive & Consistently Successful Study Material & Test Series Module, spanning across
both Physical and Online Programs in the entire Country. As a result, LegalEdge was able to engineer Clean-Sweep-
Landslide figures of a handsome 64 Selections under 100 ranks, and a whopping 273 selections under 500 ranks in
CLAT 2021. With AILET being no different, a total of 34 of our students found their way into NLU, Delhi in 2021.
In a nutshell, every second admit in a Top National Law School in 2021 came from the LegalEdge Preparation
Ecosystem.

ALL INDIA LAW ENTRANCE TEST 2023


MOCK AILET #02

Scan this code after the test


Duration : 90 Minutes Candidate Name : _____________
Max. Marks : 150 Batch : _____________
Centre Name : __________ Contact No. : _____________

to punch in your answers


INSTRUCTIONS TO CANDIDATES (Test ID: 24902)

1. No clarification on the question paper can be sought. Answer the questions as they are.
2. There are 150 multiple choice objective type questions.
3. Each question carries ONE mark. Total marks are 150.
4. There is a negative marking of 0.25 marks for every incorrect answer.
5. Candidates have to indicate the correct answer by darkening one of the four responses provided, with a BALL PEN (BLUE
OR BLACK) in the OMR Answer Sheet.
Example: For the question, "Where is the TajMahal located?", the correct answer is (b).
The candidate has to darken the corresponding circle as indicated below :
(a) Kolkata (b) Agra (c) Bhopal (d) Delhi
Right Method Wrong Methods

6. Answering the questions by any method other than the method indicated above shall be considered incorrect and no marks
will be awarded for the same.
7. More than one response to a question shall be counted as wrong.
8. The candidate shall not write anything on the OMR Answer Sheet other than the details required and, in the spaces, provided
for.
9. After the test is over, the candidate should keep the OMR Answer Sheet and Test Paper and can carry it back with
themselves.
10. The use of any unfair means by any candidate shall result in the cancellation of his/her candidature.
11. Impersonation is an offence and the candidate, apart from disqualification, may have to face criminal prosecution.
12. The candidates shall not leave the hall before the Test is over.
13. For the assessment of your answers, the students are requested to carry smartphone with working data connection,
to punch in your answers after the test on our portal.
Students cannot leave the answers hall without punching their answers.
SECTION-A : ENGLISH LANGUAGE

Directions(Q.1-Q.3): There are two blanks in each of the following sentences. From the pairs of words given
below each sentence, choose the pair that fills the blanks most appropriately.

1. The notion of ______ with the genes of an entire species inevitably worries bioethicists, but many experts believe
that mosquito born diseases are so dangerous that we have no choice but to try — at least in the most _______
countries with the least effective control systems.
(a) adjusting. . . lethal (b) adapting... populated
(c) redesigning. . backward (d) tampering .. . vulnerable

2. Kathakali lovers gathered at a typical ______ can’t wait to see the drama unfold, but they are hardly aware of
the literal pain of the artists behind their ______ masks.
(a) program. . . jazzy (b) entertainment. . . flashy
(c) performance . .. colorful (d) skit. . . showy

3. There is no glory in war _____ the blood it _______


(a) thinking.., demands (b) considering . . . sheds
(c) worth . . . costs (d) comparing .. . spills

Directions(Q.4-Q.5): The following questions have a paragraph from which the last sentence has been
deleted. From the given options, choose the one that completes the paragraph in the most appropriate way.

4. Acid attack is a form of gender-based violence, which in turn is a violation of human rights. The UN Convention
on the Elimination of All forms of Discrimination against Women (CEDAW), ratified by the Indian government,
prohibits -violence that is directed against a woman because she is a woman or that affects women
disproportionately”, resulting in perpetuation of gender inequality and discrimination. Sally Engle Merry,
professor of anthropology at New York University and faculty director of the Centre for Human Rights and
Global Justice at the New York University School of Law, notes in her book Human Rights and Gender Violence:
Translating International Law into Local Justice, “Because violence against women refers to bodily injury as do
other human rights violations such as torture, it is relatively a straightforward violation. Like torture, it is about
injury, pain and death. Moreover because gender violence is deeply embedded in systems of kinship, religion,
warfare and nationalism, its prevention requires major social changes in communities, families and nations.”
_______
(a) In the case of acid attacks, the term ‘survivor’ is preferred over the passive ‘victim’ by those who have
lived through the attack.
(b) Whatever said and done, in many parts of the world, violence against women appears to be an everyday,
normal problem rather than a violation of human rights.
(c) With the addition of Section 326A in the Indian Penal Code in 2013, acid throwing was given the status of
a cognizable and non-bailable offence.
(d) Proper medical care needs to be ensured and rehabilitative measures put in place for bringing back the
affected into me mainstream.

5. A sharp slide in solar power tariffs from a high of nearly Rs.18 a unit in 2010 to a record low of Rs.3 is not an
unmixed blessing for this up-and-coming sector. In fact, the downside of the price crash may in same respects
outweigh the anticipated gains. Though moderation in prices was vital to make solar power competitive against
other forms of conventional and non-conventional power, a price crash of this order without concomitant
advances in basic technology may prove baneful. Theoretically, cheaper costs can make solar power one of the
key forms of energy in the future with positive environmental implications. Besides, it may boost demand for
solar power as it would come in handy for power distribution companies to augment supplies to domestic and

Head Office: 127, Zone II, MP Nagar, Bhopal |+91-7676564400| https://www.toprankers.com Page 2 of 36
agricultural consumers even at the government-imposed low charges rather than resorting to power shedding.
_______.
(a) Competitive cost and the economic viability of the projects are prerequisites for the smooth growth of the
solar energy sector
(b) Banks are already turning wary of lending to projects which offer meagre returns or seem financially
unviable,
(c) Even though solar plants don’t have any substantial recurring costs, they certainly incur maintenance
expenses.
(d) A noteworthy point is that the tariff fall is not entirely the outcome of enhancement in the efficiency of
converting solar radiation into electricity, though admittedly, the cost of solar equipment has been
dropping due to gradual improvements in the manufacturing technology.

Directions(Q.6-Q.10): Read the given passages and answer the questions that follow them.
It’s a mystery that presented itself unexpectedly: The radioactive decay of some elements sitting quietly in
laboratories on Earth seemed to be influenced by activities inside the Sun, 93 million miles away.
Is this possible?
Researchers from Stanford and Purdue University believe it is. But their explanation of how it happens opens
the door to yet another mystery.
There is even an outside chance that this unexpected effect is brought about by a previously unknown particle
emitted by the sun. “That would be truly remarkable,” said Peter Sturrock, Stanford professor emeritus of applied
physics and an expert on the inner workings of the Sun.
The story begins, in a sense, in classrooms around the world, where students are taught that the rate of decay of
a specific radioactive material is a constant. This concept is relied upon, for example, when anthropologists use
carbon-14 to date ancient artifacts and when doctors determine the proper dose of radioactivity to treat a cancer
patient.
But that assumption was challenged in an unexpected way by a group of researchers from Purdue University
who at the time were more interested in random numbers than nuclear decay (Scientists use long strings of
random numbers for a variety of calculations, but they are difficult to produce. since the process used to produce
the numbers has an influence on the outcome.)
Ephraim Fischbach, a physics professor at Purdue, was looking into the rate of radioactive decay of several
isotopes as a possible source of random numbers generated without any human input. (A lump of radioactive
cesium-137. for example. may decay at a steady rate overall, but individual atoms within the lump will decay in
an unpredictable, random pattern. Thus the timing of the random ticks of a Geiger counter placed near the cesium
might be used to generate random numbers.)
As the researchers pored through published data on specific isotopes, they found disagreement in the measured
decay rates — odd for supposed physical constants.
Checking data collected at Brookhaven National Laboratory on Long Island and the Federal Physical and
Technical Institute in Germany, they came across something even more surprising: long-term observation of the
decay rate of silicon-32 and radium-226 seemed to show a small seasonal variation. The decay rate was ever so
slightly faster in winter than in summer.
Was this fluctuation real, or was it merely a glitch in the equipment used to measure the decay, induced by the
change of seasons, with the accompanying changes in temperature and humidity?
“Everyone thought it must be due to experimental mistakes, because we’re all brought up to believe that decay
rates are constant.” Sturrock said.
On Dec 13, 2006, the Sun itself provided a crucial clue, when a solar flare sent a stream of particles and radiation
toward Earth. Purdue nuclear engineer Jere Jenkins, while measuring the decay rate of manganese-54, a short-
lived isotope used in medical diagnostics, noticed that the rate dropped slightly during the flare, a decrease that
started about a day and a half before the flare.
If this apparent relationship between flares and decay rates proves true, it Could lead to a method of predicting
solar flares prior to their occurrence, which could help prevent damage to satellites and electric grids, as well as
save the lives of astronauts in space.
Head Office: 127, Zone II, MP Nagar, Bhopal |+91-7676564400| https://www.toprankers.com Page 3 of 36
The decay-rate aberrations that Jenkins noticed occurred during the middle of the night in Indiana — meaning
that something produced by the sun had traveled all the way through the Earth to reach Jenkins” detectors. What
could the flare send forth that could have such an effect?
Jenkins and Fischbach guessed that the culprits in this bit of decay-rate mischief were probably solar neutrinos,
the almost weightless particles famous for flying at almost the speed of light through the physical world —
humans, rocks, oceans or planets — with virtually no interaction with anything.
Then, in a series of papers published in Astroparticle Physics, Nuclear Instruments and Methods in Physics
Research and Space Science Reviews, Jenkins, Fischbach and their colleagues showed that the observed
variations in decay rates were highly unlikely to have come from environmental influences on me detection
systems.
Their findings strengthened the argument that the strange swings in decay rates were caused by neutrinos from
the Sun. The swings seemed to be in sync with the Earth”s elliptical orbit, with the decay rates oscillating as the
Earth came closer to the Sun (where it would be exposed to more neutrinos) and then moving away.
So there was good reason to suspect the Sun, but could it be proved?

6. According to the passage. -another mystery” that has presented itself is the
(a) influence of temperature and humidity on the decay rate of radioactive elements.
(b) “mischief” caused by solar neutrinos.
(c) “strange” variation in decay rates.
(d) linkage between solar flares and the inner life of radioactive elements on Earth.

7. According to Fischbach, the rate of radioactive decay


(a) in a lump of cesium-137 is constant.
(b) in individual atoms within a cesium-137 lump is constant
(c) in individual atoms within a cesium-137 lump is variable.
(d) in a lump of cesium-137 is variable.

8. The “even more surprising” observation by the researchers was that


(a) there was disagreement in the measured decay rates of cesium-137.
(b) the decay rates of two specific isotopes were higher in winter than in summer.
(c) the decay rates of two specific isotopes were higher in summer than winter.
(d) the decay rates of two specific isotopes were higher due to glitch in the equipment used to measure the
decay.

9. According to the passage, the decay rate of managanese-54 dropped ‘before’ the flare because
(a) solar neutrinos are weightless and can travel faster than light.
(b) the solar flare took some time to reach the Earth.
(c) the flare may have sent forth some particles that affected the isotope.
(d) the flare may have affected the environment in an undetected manner.

10. According to the passage, the decay rates of some radioactive isotopes _______ as the Earth comes closer to the
sun.
(a) stabilize (b) increase (c) swing (d) decrease

Head Office: 127, Zone II, MP Nagar, Bhopal |+91-7676564400| https://www.toprankers.com Page 4 of 36
Directions(Q.11-Q.13): In each of the questions below, four different ways or writing a sentence are indicated
Choose the best way for writing the sentence.

11. (a) A Rufous-tailed Rock Thrush (Monticola saxitilis), considered an autumn passage migrant bird species in
Pakistan and India, was first seen and photographed by an expedition last year near the Shey monastery
within the Shey-Phoksundo National Park.
(b) A Rufous-tailed Rock Thrush (Monticola saxitilis), considered a autumn passage migrant bird species in
Pakistan and India, was first seen and photographed by an expedition last year near the Shey monastery
within the Shey-Phoksundo National Park.
(c) A Rufous-tailed Rock Thrush (Monticola saxitilis), considered an autumn passage emigrant bird species in
Pakistan and India, was first seen and photographed by an expedition last year near the Shey monastery
within the Shey-Phoksundo National Park.
(d) A Rufous-tailed Rock Thrush (Monticola saxitilis), considered an autumn passage migrant bird species in
Pakistan and India, was first saw and photographed by an expedition last year near the Shey monastery within
the Shey-Phoksundo National Park.

12. (a) Basi, the world’s older captive panda with a celebrity status among the endangered species, celebrated her
37th birthday on Wednesday, equivalent to more than 100 human years in east China’s Fujian Province.
(b) Basi, the world’s oldest captive panda with a celebrity status between the endangered species, celebrated her
37th birthday on Wednesday, equivalent to more than 100 human years in east China’s Fujian Province.
(c) Basi, the world’s oldest captive panda with a celebrity status among the endangered species, celebrated her
37th birthday on Wednesday, equivalent to more than 100 human years in east China’s Fujian Province.
(d) Basi, the world’s oldest captive panda with a celebrity status among the endangered species, celebrated her
37th birthday on Wednesday, equivalent to more than 100 human ears in east China’s Fujian Province.

13. (a) Fondly called as ‘cutest falcon’ among the birdwatchers, Amur Falcon an small raptor that breeds in south-
eastern Siberia and Northern China was spotted for the first time in Central India with photographic
authentication.
(b) Fondly called as ‘cutest falcon’ among the birdwatchers, Amur Falcon a small raptor that breeds in south-
eastern Siberia and Northern China was spot for the first time in Central India with photographic
authentication.
(c) Fondly called as ‘cutest falcon’ among the birdwatchers. Amur Falcon a small raptor that breeds in south-
eastern Siberia and Northern China was spotted for the first time in Central India with photographic
authenticity.
(d) Fondly called as ‘cutest falcon’ among the birdwatchers, Amur Falcon a small raptor that breeds in south-
eastern Siberia and Northern China was spotted for the first time in Central India with photographic
authentication.

Directions(Q.14 and Q.15): There are two blanks in each of the following sentences. From the pairs of words
given below each sentence, choose the pair that fills the blanks most appropriately.

14. The dace is a lively, active fish, of ____ habits, and exceedingly _____, depositing its eggs in May and June at
the roots of aquatic plants or in the gravelly beds of the streams it frequents.
(a) convivial ... unfruitful (b) vivacious ... sterile
(c) solitary...fecund (d) gregarious ... prolific

15. The worst ____ of all, defiling the Host, is mentioned frequently and generally brought the death penalty
accompanied by the cruelest and most ______ tortures.
(a) blasphemy ... worthy (b) piety .. despicable
(c) sacrilege ... ignominious (d) profanity ... respectable

Head Office: 127, Zone II, MP Nagar, Bhopal |+91-7676564400| https://www.toprankers.com Page 5 of 36
Directions(Q.16-Q.19): The sentences given in each of the following questions, when properly sequenced, form
a coherent paragraph. Each sentence is labelled with a letter. From among the four choices given below each
question, choose the most logical order of sentences that constructs a coherent paragraph.

16. A. The survey is the final piece in a jigsaw which shows that, against expectations, the shake-up in higher
education of the past few years has failed to dent the image of a traditional university education.
B. A national survey shows the urge to fly from the family home to study for a degree is stronger now than it
ever was.
C. Britain’s undergraduates are continuing to study away from their home towns despite the revolution in higher
education, which has seen fees rise to as much as £9,000 a year.
D. Figures contained in it show that just one in 20 prospective students — those now in the sixth-form —
wishes to study somewhere close to home.
(a) CABD (b) CBDA (c) BCDA (d) BDAC

17. A. Compared with the Latvian capital Riga, which shares the honour in 2014, Umea might seem an unusual
choice.
B. For the first time since 1888, when a fire turned hundreds of houses to ash, Umea is hot property.
C. But with one of the world’s best design schools, a rocking music scene and a story that’s been shaped by
the Russians and the Sami — the indigenous population of the Arctic — it’s every bit as captivating.
D. Sweden’s eleventh-biggest city, 400 miles north of Stockholm amid a sea of pine, is next year’s European
Capital of Culture.
(a) BCDA (b) DACB (c) BDAC (d) DBAC

18. A. It is therefore natural that the Kyrgez Republic has traditionally been home to nomadic tribespeople who
kept moving from pasture to pasture with their portable yurts (cloth tents) and livestock.
B. Almost 80 per cent of the country is mountainous but the hills are interspersed with expansive green pastures
and lush valleys.
C. Sandwiched between the endless steppes of Kazakhstan and the barren desert landscape that makes up most
of Uzbekistan, the Kyrgyz Republic is a haven of verdure.
D. Arable land is limited to the fertile Ferghana Valley in the south, shared with neighbouring Uzbekistan, and
comprises less than 8% of the country’s area.
(a) CABD (b) CBDA (c) CADB (d) CDAB

19. A. But the government has to identify a way to balance development with conservation, as India’s tigers are on
the brink of extinction and any delay in conservation efforts will almost certainly result in our losing this
species.
B. However, in the light of the precariousness of the tiger’s existence, there is a need for some lateral thinking
in tiger conservation.
C. On the positive side, tiger conservation is not rocket science; the government just needs to persistently follow
the extant conservation strategies with renewed vigour and determination and perform periodic management-
effectiveness evaluations.
D. While the country’s government is not consciously against tiger conservation, the reality of population
pressure puts it under tremendous compulsion to sometimes sideline conservation in favour of more pressing
developmental and livelihood schemes.
(a) DABC (b) CDBA (c) DACB (d) CDAB

Head Office: 127, Zone II, MP Nagar, Bhopal |+91-7676564400| https://www.toprankers.com Page 6 of 36
Directions(Q.20-Q.24): In each question, there are five sentences or parts of sentences that form a paragraph.
Identify the sentence(s) or part(s) of sentence(s) that is/are correct in terms of grammar and usage. Then,
choose the most appropriate option.

20. A. One way to celebrate Darwin’s birthday is contemplating how evolutionary studies
B. can achieve broader adoption in secondary and higher education.
C. Natural selection and the complementary idea or how genes, individuals and species change
D. over time should be as much part of developing critical
E. thinking skills as deductive reasoning and study of ethics.
(a) A, B and D (b) A, D and C (c) B and C (d) D and E

21. A. The psychological pressure looks well orchestrated and is being well
B. executed, though it would gain even great strength if American intelligence
C. could lead the inspectors to another damning evidence.
D. Such knowledge may be being held back to disclose at a time when war
E. could plausibly be launched at a moments notice and thus when the dictator would have no room to
wriggle.
(a) A, C and E (b) Only A (c) B and D (d) C, D and E

22. A. Digital technologies are not only making it easier to copy all sorts of works,
B. but also reducing sharply the costs of creating or distributing them,
C. and also reducing the required incentives. The flood of free
D. content on the internet has showed that most creators do not
E. need incentives that stretch across generations.
(a) A and E (b) B, D and E (c) B, C and D (d) B and E

23. A. Spring has sprang. The hills north of Beijing are alive
B. with the sound of noisy restaurant attendants, some waiving red banners,
C. standing at the side of the road shouting, ‘Stop here for a delicious meal!”
D. at the mobs of city dwellers zooming by in their cars.
E. Car ownership more than tripled in the last four years, making China the world’s second largest market
after the U.S.
(a) A, B and D (b) C and E (c) A and E (d) B, C and E

24. A. One of the driest springs ever recorded in northern Europe could lead to power blackouts this summer,
B. with nuclear reactors going of line because of low river levels.
C. The exceptionally dry weather will also rise food prices and has already forced water restrictions
D. among millions of people, say, governments, farm groups and meteorological organizations across the
continent.
E. Large parts of northern and eastern European countries have had their driest three month spells in more
than 50 years.
(a) Only A (b) B, C and D (c) B and E (d) A and E

25. In the given questions, a word has been given and there are three ways in which the word has been used, in
similar or different forms. You need to see which of the sentences have correctly used the highlighted word, and
mark that as your answer.
Nature
1. He has always taken advantage of his brother's simple-minded nature.
2. The laws of nature are not to be meddled with.
3. The nature of this job is such that you will have to put in a lot of effort.
(a) Only 2 (b) Only 1 and 2 (c) Only 1 and 3 (d) All 1, 2 and 3
Head Office: 127, Zone II, MP Nagar, Bhopal |+91-7676564400| https://www.toprankers.com Page 7 of 36
26. In the given questions, a word has been given and there are three ways in which the word has been used, in
similar or different forms. You need to see which of the sentences have correctly used the highlighted word, and
mark that as your answer.
Critical
1. His courage and perseverance at this critical time were of inestimable value.
2. Temperature is a critical factor in successful fruit storage.
3. She is too critical of people and considers them inferior to herself.
(a) Only 3 (b) Only 1 and 2 (c) Only 1 and 3 (d) All 1, 2 and 3
27. In the given sentences, a blank is given indicating that something is missing. From the given four options a
combination of words would fit the blank thereby making it grammatically and contextually correct.
For now, however, visitors can search this underwater city for a hidden entrance - if not to the underworld,
then at least to some ___________ subterranean treasures.
A. Spectacular
B. Picturesque
C. Dazzle
D. Sensation
(a) BD (b) AC (c) AB (d) CD
28. In the given sentences, a blank is given indicating that something is missing. From the given four options a
combination of word would fit the blank thereby making it grammatically and contextually correct.
In recent years, he spoke often ____________ about exploring space more urgently than we humans are now
doing.
A. Eloquently
B. Surely
C. Persuasively
D. Distinctive
(a) AB (b) BC (c) AD (d) AC
29. In each of the following questions, a sentence is given with the phrase or idiom highlighted in bold. Select the
option given below that cannot replace the phrase in bold and mark that as your answer.
Such stars burn hydrogen such slowly that they could keep us warm for ten trillion years, about a thousand
times longer than the lifetime of the sun.
(a) at a snail's pace (b) at such a slow rate
(c) so gradually (d) All can replace
30. In each of the following questions, a sentence is given with the phrase or idiom highlighted in bold. Select the
option given below that cannot replace the phrase in bold and mark that as your answer.
Many of these exoplanets have been too distant to learned much about.
(a) gain much information about
(b) learn much about
(c) get insights from
(d) All can replace
31. In each of the following questions, a sentence is given with the phrase or idiom highlighted in bold. Select the
option given below that cannot replace the phrase in bold and mark that as your answer.
It's shocking that publishers aren't making much money, giving how much time the public spends consuming
their products.
(a) taking into account how
(b) considering what
(c) especially in the light of how
(d) bearing in mind how

Head Office: 127, Zone II, MP Nagar, Bhopal |+91-7676564400| https://www.toprankers.com Page 8 of 36
Directions (Q.32-Q.36): In the following passage, certain words are either underlined or in bold. Some of these
words in bold are incorrectly used either contextually or grammatically. You are provided with four options (a),
(b) and (c) one of which can replace the word in bold in order to make the sentence coherent and meaningful. If
the word is correctly used in the paragraph and no correction is required then mark “No Improvement “as your
answer.
When World War I broke out across Europe in 1914, President Woodrow Wilson (A) proclaimed the United
States would remain neutral, and many Americans supported this policy of non-intervention. However, public
(B) attitude about neutrality started to change after the sinking of the British ocean liner Lusitania by a German
U-boat in 1915; almost 2,000 people (C) cherished, including 128 Americans. Along with news of the
Zimmerman telegram threatening an (D) link between Germany and Mexico, Wilson asked Congress for a
declaration of war against Germany. The U.S. officially entered the (E) convict on April 6, 1917.

32. A
(a) ensnared (b) reclaimed (c) defiled (d) No Improvement

33. B
(a) opinion (b) thinking (c) position (d) No Improvement

34. C
(a) replenished (b) perished (c) relished (d) No Improvement

35. D
(a) amalgamation (b) associate (c) alliance (d) No Improvement

36. E
(a) edict (b) indict (c) afflict (d) conflict

37. Read each sentence to find out whether there is any grammatical error in it. The error, if any, will be in one part
or more of the sentence. Choose the option with parts with no errors as your answer.
While waiting for my train to take me back (A)/ to the city, we got talking about the importance (B)/ of being
satisfied with what we have and (C)/ not took unnecessary risks to make more money (D)/ when we already
have enough. (E)
(a) ABCE (b) ACDE (c) ABDE (d) Other than these options

38. Read each sentence to find out whether there is any grammatical error in it. The error, if any, will be in one part
or more of the sentence. Choose the option with parts with no errors as your answer.
People often measure success (A)/ in terms of money and ignore (B)/ other measures of success like (C)/
personal reputation, a happy (D)/ family life and well health. (E)
(a) ABCE (b) ACDE (c) BCDE (d) Other than these options

39. In the question given below, a sentence is given with one blank, followed by five options, each having a word
which may or may not fit in the blank. From the given options, choose the one that would correctly fit in the
given blank.
When people underestimate us, they saddle us with negative assumptions that can be ____ and can make us
question our own abilities.
(a) disrupt (b) beneficial (c) sustainable (d) destabilising

Head Office: 127, Zone II, MP Nagar, Bhopal |+91-7676564400| https://www.toprankers.com Page 9 of 36
40. In the question given below, a sentence is given with one blank, followed by five options, each having a word
which may or may not fit in the blank. From the given options, choose the one that would correctly fit in the
given blank.
Most people want to believe that working out and fat loss needs to be hard, but a basic program performed
____ can bring you a really long way.
(a) erratically (b) consistently (c) impartial (d) occasionally

41. In the question given below, a sentence is given with one blank, followed by five options, each having a word
which may or may not fit in the blank. From the given options, choose the one that would correctly fit in the
given blank.
Thinking of talent as innate makes our world more manageable, more comfortable as it relieves a person of the
_____ of expectation.
(a) responsible (b) assistance (c) blessing (d) burden

42. In the question given below, a sentence is given with one blank, followed by five options, each having a word
which may or may not fit in the blank. From the given options, choose the one that would correctly fit in the
given blank.
The Lebanese army raided the boat and _____ dozens of water bottles, tuna cans, packs of bread, boxes of
Picon cheese, and life jackets.
(a) invention (b) finding (c) locate (d) discovered

Directions(Q.43-Q.47): Read the passage carefully and answer the questions.


This notion of presence as the antidote to fear and the crucible of love is as old as the human heart, as old as the
consciousness that first felt the blade of anticipatory loss pressed against the exposed underbelly of the longing
for connection. It is at the centre of millennia-old Buddhist philosophy and comes alive afresh, in a splendidly
practical way, in Fear: Essential Wisdom for Getting Through the Storm by the great Vietnamese Buddhist
teacher and peace activist Thich Nhat Hanh, whose teachings continue to enrich, ennoble, and empower. The
general Buddhist style of befriending complexity is through simplicity and with a particular gift for simple words
strung into a rosary of immense wisdom radiating immense kindness.
Unlike a prayer — which channels a hope at some imagined entity capable of interceding in favour of that hope
and has only as a side benefit, the psychological self-clarification that comes from honing our hopes in language
— a mantra is not addressed at anything or anyone external and is entirely devoted to distilling the object of
hope to its clearest essence. This, in and of itself, transforms the hope into an intent, making it more actionable
— but also saving it from the particular complacency against which Hanh admonished as he considered the vital
relationship between fear and hope. A mantra is therefore not a form of magical thinking, for while there is a
sense of magic to how such distillation seems to shift the situation by its very utterance, it is an entirely practical
sort of magic, for a mantra simply clarifies, concentrates, and consecrates intent, and all meaningful
transformation springs from purposeful, devoted intent.
Simple though this mantra might seem, Hanh reminds us that actually cultivating the capacity for it — the
capacity for presence, which is where our capacity for love resides — is intensely difficult against the tidal wave
of demand and distraction that sweeps everyday life and sweeps us along with it, leaving us always on the brink
of drowning, bereft of what Emerson celebrated as “the power to swell the moment from the resources of our
own heart until it supersedes sun & moon & solar system in its expanding immensity.” A century after Tolstoy
insisted that “love is a present activity only,” Hanh gently reminds us that the greatest resource of our own heart
— our greatest source of power, our mightiest antidote to fear — is the quality of love we give through the
quality of our presence.
In a sentiment of especial relevance and consolation in these disembodied times, Hanh reminds us that these
mantras can be performed across distance, across wires and cables and screens, not requiring the physical
presence of the beloved — however they are articulated, they are at bottom meditations containing all four
elements of true love as described by the Buddha: love, compassion, joy, and freedom. While the third mantra,

Head Office: 127, Zone II, MP Nagar, Bhopal |+91-7676564400| https://www.toprankers.com Page 10 of 36
“Mantra for Relieving Suffering,” could be magnified and deepened by the atomic rewards of Thich Nhat Hanh’s
“hugging meditation,” it too can be extended across the digital distance.

43. Which of the accompanying is the opinion of Hanh on the subject of mantra and prayer?
A. Mantra converts hope into an aim, making it more practical than prayer.
B. Prayer directs a faith towards some imagined institution worthy of interceding in that hope's favour.
C. Mantra is a magic that is practical as it simplifies, focuses, and consecrates intent, and all significant
transformation stems from deliberate, devoted intent.
(a) Only A and B (b) Only B and C (c) Only A and C (d) A, B and C

44. How does Hanh’s definition of love differ from Tolstoy?


(a) Hans reminds us that the biggest asset of our own heart, our biggest source of strength, our fiercest remedy
for fear is the degree of love we provide through the quality of our presence whereas Tolstoy argued that
love is a current activity alone.
(b) Hans reminds us that the shortest asset of our own heart, our lowest source of strength, our weakest remedy
for fear is the degree of love we provide through the quality of our presence whereas Tolstoy argued that
love is a current activity alone.
(c) Hanh reminds us that building the capacity for it — the capacity for presence, which is where our potential
for love lives — is extremely challenging in the face of the tidal wave of demand and distraction that sweeps
everyday life and sweeps us along with it but Tolstoy argued that love is a current activity alone.
(d) None of the above.

45. Which of the accompanying is NOT an argument given by Hanh?


(a) The mantras can be performed devoid of the physical presence.
(b) Buddha said that the four elements of true love are love, empathy, happiness, and independence.
(c) The four mantras are affected by distance and separation as their true essence is lost.
(d) The microscopic rewards of Hanh's hugging meditation can magnify and deepen the third mantra.

46. Which of the accompanying throws light on the techniques used by Buddhists to embrace intricacy?
(a) Prayer, which brings forth a sense of a spiritual or loving presence and alignment with God or an
immersion into a universal unconsciousness.
(b) Letting go of complicated matters and things that are beyond our control.
(c) Easiness and an ability to string together simple words into a thread of tremendous wisdom exuding
enormous kindness.
(d) None of the above.

47. Why did Hanh tell us that the simplicity of mantra is just a facade?
(a) The idea of presence as an antidote to fear and a testing ground for love is extremely old as the
consciousness that first felt the blade of expected loss pressed against the need for connection.
(b) A mantra is directed at nothing or no one outside of oneself and is solely focused on refining the object of
hope to its purest form.
(c) Both A and B.
(d) Fostering the potential for mantra, the ability for involvement, which is where our capacity for love resides
is extremely difficult in the face of the various demand and diversion that envelops our life.

48. Select the most appropriate synonym of the given word.


INIMICAL
(a) comparable (b) deliberate (c) influential (d) hostile

Head Office: 127, Zone II, MP Nagar, Bhopal |+91-7676564400| https://www.toprankers.com Page 11 of 36
49. Select the most appropriate antonym of the given word.
PRECLUDE
(a) assist (b) deter (c) froward (d) fervent

50. In each of the questions given below, a sentence is given with one blank, followed by 4 options, each having a
word which may or may not fit in the blank. From the given options, choose the one that gives the correct word
that fits in the blank.
The increase in global population combined with increased economic prosperity in emerging economies, the
demand for meat and dairy food products is set to rapidly increase and such an/a _____ demand for animal
food products simply cannot be met by traditional farming methods.
(a) placid (b) insatiable (c) squeamish (d) quest

Head Office: 127, Zone II, MP Nagar, Bhopal |+91-7676564400| https://www.toprankers.com Page 12 of 36
SECTION B: CURRENT AFFAIRS & GENERAL KNOWLEDGE

51. The 34 km rail line between India and Nepal was recently given a green nod. The line will operate between
which of the following cities?
(a) Jainagar and Kurtha (b) Jainagar and Janakpur dham
(c) Janakpur dham and Kurtha (d) Bijalpura to Kurtha

52. Consider the following about Transparency International:


I. It is an international non -governmental organisation founded in 1993 based in Berlin, Germany.
II. The CPI shows that control of corruption has stagnated or worsened in 86% of countries over the last decade.
Which of the above is/are true?
(a) Only I (b) Only II (c) Both I and II (d) Neither I nor II

53. Who among the following has won the government’s disinvestment bid for Air India?
(a) Talace Pvt. Ltd (b) Reliance Pvt. Ltd
(c) Agrico Pvt. Ltd (d) Tanla Pvt. Ltd

54. The idea of One Sun One World One Grid was first introduced in which of the following years?
(a) 2015 (b) 2016 (c) 2017 (d) 2018

55. Who among the following released the Pandora Paper Leaks?
(a) International Consortium of Investigative Journalists
(b) Centre for Public integrity
(c) Investigative Reporters & Editors
(d) The Center for Investigative Reporting

56. How many PM Mitra Parks have been announced to be set up by the Union Government?
(a) 5 (b) 7 (c) 9 (d) 11

57. Power Finance Corporation has become the ______ public sector enterprise to get the ‘Maharatna’ status in the
country.
(a) 9th (b) 10th (c) 11th (d) 12th

58. Water Day is observed on which of the following days?


(a) 22nd March (b) 21st December (c) 1st July (d) 31st July

59. Human Development Index is released by which of the following organization?


(a) United Nations Development Program
(b) United Nations Educational, Scientific and Cultural Organization
(c) United Nations Industrial Development Organization
(d) World Bank Group

60. Which of the following is one of the objectives of AMRUT Mission?


(a) It will provide 100% coverage of sewerage and septage in 500 AMRUT cities.
(b) Recycling and reuse of treated wastewater is expected to cater to 20% of total water needs of the cities and
40% of industrial demand
(c) It will build upon the progress of AMRUT to address water needs, rejuvenate water bodies, better manage
aquifers, reuse treated wastewater, thereby promoting a circular economy of water.
(d) All of the above

Head Office: 127, Zone II, MP Nagar, Bhopal |+91-7676564400| https://www.toprankers.com Page 13 of 36
61. The Intergovernmental Panel on Climate Change was set up in which of the following years?
(a) 1988 (b) 1990 (c) 1992 (d) 1994

62. The CBI comes under which of the following Ministries?


(a) Ministry of Home Affairs
(b) Ministry of Defense
(c) Ministry of Personnel, Public Grievances and Pensions.
(d) Ministry of Planning

63. Who represented India at the G20 Sherpa Meet 2021?


(a) Nirmala Sitharaman (b) Piyush Goyal
(c) S. Jaishankar (d) Meenakshi Lekhi

64. Who among the following is the Permanent Representative of India to the United Nations?
(a) T.S. Tirumurti (b) Hardeep Singh Puri
(c) Asoke Kumar Mukerji (d) Syed Akbaruddin

65. Who among the following has been appointed as the first President of Barbados?
(a) Edmund Hinkson (b) Arthur Holder
(c) Mia Mottley (d) Sandra Prunella Mason

66. The Jaitpur Project is a key component of the strategic partnership between India and which country?
(a) USA (b) Japan (c) Germany (d) France

67. The Uighurs are a predominantly Muslim minority ethnic group of which of the following Muslim Nation?
(a) Turkey (b) UAE (c) Iraq (d) Egypt

68. How many Freedoms of Air does the International Civil Aviation Organisation recognises?
(a) three (b) Four (c) Five (d) Six

69. Which of the following is India’s rank in the Henley Passport Index, 2022?
(a) 81st (b) 83rd (c) 85th (d) 86th

70. Who among the following has been appointed as the Chairman of the Indian Space Research Organisation?
(a) Dr. K. Sivan (b) Dr. AS Kirankumar
(c) Dr. Somanath (d) Dr. K. Radhakrishnan

71. Which of the following sites has been finalised to reintroduce Cheetah in India?
(a) Kuno Palpur National Park
(b) Nauradehi Wildlife Sanctuary
(c) Gandhi Sagar Wildlife Sanctuary
(d) Mukundara Tiger Reserve

72. Consider the following about the Green Energy Corridor:


I. It will facilitate grid integration and power evacuation of approximately 20 GW of Renewable Energy (RE)
power projects in seven States namely, Gujarat, Himachal Pradesh, Karnataka, Kerala, Rajasthan, Tamil
Nadu and Uttar Pradesh.
II. The transmission systems will be created over a period of five year from Financial Year 2021-22 to 2025-
Which of the above is/are TRUE?
(a) Only I (b) Only II (c) Both I and II (d) Neither I nor II

Head Office: 127, Zone II, MP Nagar, Bhopal |+91-7676564400| https://www.toprankers.com Page 14 of 36
73. The Rajya Sabha has passed the Constitution (Scheduled Tribes) Order (Amendment) Bill, 2021. The bill
provides for modifying which of the following Part of Schedule to the Constitution (Scheduled Tribes) Order,
1950, relating to the state of Arunachal Pradesh.?
(a) Part VIII (b) Part XVIII (c) Part XV (d) Part VI

74. NGT has directed Punjab and Rajasthan governments to submit compliance reports about remedial action being
taken to curb the inflow of effluent discharge into which of the following rivers?
(a) Gomti and Mahi (b) Luni and Banas
(c) Sutlej and Beas (d) Sutlej and Gomti

75. A Supreme Court-appointed committee has questioned relaxations given for the upgrade of a 4.7-km road
(Laldhang-Chillarkhal road) in the buffer zone of Rajaji Tiger Reserve, which is in the state of?
(a) Uttar Pradesh (b) Uttrakhand (c) Gujarat (d) Chattisgarh

76. Judima, a home-made rice wine of Dimasa tribe has become the first traditional brew in all of northeast to bag a
Geographical Indication (GI) tag. It is from which of the following state?
(a) Assam (b) Manipur (c) Arunachal Pradesh (d) Sikkim

77. The Election Commission of India (ECI) has decided to freeze the election symbol of which of the following
party?
(a) Nationalist Congress Party
(b) CPI R
(c) Dravida Munnetra Kazhagam
(d) Lok Janshakti Party

78. According to a ruling of the High Court, Transgenders are to appear in ____ Police Recruitment Exam. The State
government responded to the same and added transgender column in online application form.
What should replace the above blank?
(a) Kerala (b) Tamil Nadu (c) Kolkata (d) Delhi

79. The Government has notified new rules under the Medical Termination of Pregnancy (Amendment) Act, 2021.
The gestational limit for termination of a pregnancy has been increased from 20 to __ weeks for certain categories
of women.
(a) 22 weeks (b) 24 weeks (c) 26 weeks (d) 28 weeks

80. Supreme Court (SC) has appointed an expert committee in the Pegasus case, which is to be overseen by which
of the following retired apex court judge?
(a) Justice Talukdar (b) Justice Mishra
(c) Justice Raveendran (d) Justice Malimath

Head Office: 127, Zone II, MP Nagar, Bhopal |+91-7676564400| https://www.toprankers.com Page 15 of 36
SECTION C: LOGICAL REASONING

Directions (Q.81 and Q.82): In each question below is given a statement followed by two conclusions numbered
I and II. You have to assume everything in the statement to be true, then consider the two conclusions together
and decide which of them logically follows beyond a reasonable doubt from the information given in the
statement. Give answer:

81. Statements: Russia invaded Ukraine as its representatives came back meeting United Nations' representatives
in London.
Conclusions:
1. Russia rejected the offer proposed in the meeting.
2. The meeting did not achieve its objective.
(a) If only conclusion I follow.
(b) If only conclusion II follows
(c) If neither I nor II follows and
(d) If both I and II follow.

82. Statements: Road safety statics suggest that nearly 90% of all road accidents occur at speeds below 50 Km per
hour and within 8 Km from home. Therefore, you are safer in a car if you are driving above 50Km per hour and
not within 8 Km from home.
Conclusions:
1. People tend to concentrate less when they are approaching home.
2. Most homes are in residential localities where the traffic density is high.
(a) If only conclusion I follow.
(b) If only conclusion II follows
(c) If neither I nor II follows and
(d) If both I and II follow.

83. Direction: Each question has a set of four sequentially ordered statements. Each statement can be classified as
one of the following:
Facts (F) - deal with the pieces of information that one has heard, seen or read, and which are open to discovery
or verification.
Inferences (I) - are conclusions drawn about the unknown, on the basis of the known.
Judgments (J)- are opinions that imply approval or disapproval of persons, objects, situations and occurrences
in the past, the present or the future
1. If music is the food of life, play on.
2. Rum is a drink that leaves no hangovers.
3. Computer operators are in danger of eye damage.
(a) FFF (b) JJJ (c) JFF (d) FII

84. Study the two statements labelled as Assertion (A) and Reason (R).
Assertion- The shocking number of animal cruelty cases reported every day is just the tip of the iceberg—most
cases are never reported.
Reason -The animals do not know how to raise their voice to the courts of human beings.
Point out if:
(a) Both, A and R, are true and R is the correct explanation of A
(b) Both, A and R, are true but R is not the correct explanation of A
(c) If A is true but R is false
(d) If A is false but R is true

Head Office: 127, Zone II, MP Nagar, Bhopal |+91-7676564400| https://www.toprankers.com Page 16 of 36
85. Study the two statements labelled as Assertion (A) and Reason (R).
Assertion – Studies say that the most common cause of the infection of the coronavirus is eating wild animals.
Reason- The human digestion system is incompatible with some unconventional food.
Point out if:
(a) Both, A and R, are true and R is the correct explanation of A
(b) Both, A and R, are true but R is not the correct explanation of A
(c) If A is true but R is false
(d) If A is false but R is true

86. There is a statement followed by three assumptions numbered I, II and III. You have to consider which of the
following assumptions is implicit in the statement. You have to consider which of the assumptions is implicit in
the statement.
Statement -The intervention of government is necessary for all aspects of collective human endeavour.
I. Humans are incapable of collective endeavour.
II. The government’s intervention is never harmful.
III. Collective human endeavour is unlikely to succeed without outside help.
(a) Only I is implicit. (b) Only II is implicit.
(c) Only III is implicit. (d) II and III are implicit.

87. There is a statement followed by three assumptions numbered I, II and III. You have to consider which of the
following assumptions is implicit in the statement. You have to consider which of the assumptions is implicit in
the statement.
Statement – Multinationals are expected to change the manufacturing landscape of the country.
I. Multinationals are capable of influencing the country’s manufacturing sector.
II. Multinationals have a different level of manufacturing from that used in India.
III. The multinational’s technology is superior to that used in India.
(a) Only I is implicit. (b) Only II is implicit.
(c) Only III is implicit. (d) II and III are implicit.

88. In each question below is given a statement followed by two courses of action numbered I and II. You have to
assume everything in the statement to be true and on the basis of the information given in the statement, decide
which of the suggested courses of action logically follow(s) for pursuing.
Statement – A new more fatal variant of Covid -19 is found in Europe.
Course of action-
I. The government shall ban all the flights from the European countries.
II. Mandatory Covid testing of all the passengers coming from the foreign countries at the airport shall be done.
(a) If only I follows. (b) If only II follows.
(c) If neither I nor II follows. (d) If both I and II follow.

89. In each question below is given a statement followed by two courses of action numbered I and II. You have to
assume everything in the statement to be true and on the basis of the information given in the statement, decide
which of the suggested courses of action logically follow(s) for pursuing.
Statement – No doctor could have improved my problem of indigestion till now.
Course of action –
I. I will not see any doctor now.
II. I will lose weight now to get in shape.
(a) If only I follows. (b) If only II follows.
(c) If neither I nor II follows. (d) If both I and II follow.

Head Office: 127, Zone II, MP Nagar, Bhopal |+91-7676564400| https://www.toprankers.com Page 17 of 36
90. Each question given below consists of a statement, followed by two arguments numbered I and II. You have to
decide which of the arguments is a 'strong' argument and which is a 'weak' argument.
Statement – Should India vote against Russia in the United Nations Council for attacking Ukraine?
Arguments-
I. No, Russia is an old ally of India in trade and defence systems.
II. Yes, other democracies are voting against Russia for this invasion.
Give answer:
(a) If only argument I is strong (b) If only argument II is strong
(c) If neither I nor II is strong (d) If both I and II are strong.

91. Each question given below consists of a statement, followed by two arguments numbered I and II. You have to
decide which of the arguments is a 'strong' argument and which is a 'weak' argument.
Statement – The board exams shall not be in the offline mode for this session.
Arguments-
I. As all the students are taking only online classes for the last two years in the Covid era.
II. Students have not prepared for the offline exams and having anxiety.
Give answer:
(a) If only argument I is strong (b) If only argument II is strong
(c) If neither I nor II is strong (d) If both I and II are strong.

92. Below in each of the questions are given two statements I and II. These statements may be either independent
causes or may be effects of independent causes or a common cause. One of these statements may be the effect
of the other statements. Read both the statements and decide which of the following answer choice correctly
depicts the relationship between these two statements.
Statement I– The education of medicine is cheaper in some foreign countries than in India.
Statement II- Every year thousands of students go to foreign countries for studying medicine.
Mark answer
(a) If statement I is the cause and statement II is its effect.
(b) If statement II is the cause and statement I is its effect.
(c) If both statements I and II are independent causes.
(d) If both statements I and II are effects of independent causes.

93. Choose the option which completes the given statement logically and rationally.
The art of medicine is the art of healing, not just treating, and not even just curing______
(a) Medicine should end the root cause.
(b) But the art of curing is not just the art of healing it is the art of improving.
(c) It is the first step of the medical profession that is taught.
(d) In the same way, the art of yoga is the art of meditation, not just physical exercise.

94. The Cinema Chamber Association (CCA) of a particular state instructed all the member directors and producers
to employee 80% of the workforce from the state to provide employment to local people.
Which of the following, if true, can render the instructions inoperable?
(a) Senior directors from the state, who are not the members of the association, have stopped hiring more than
20% from other states.
(b) Some of the member directors and producers have surplus manpower.
(c) The number of non-member directors and producers is more than the number of member directors and
producers.
(d) Directors and producers may incur huge losses by hiring not more than 20% from other states as there are
not many skilled workers among the local work force.

Head Office: 127, Zone II, MP Nagar, Bhopal |+91-7676564400| https://www.toprankers.com Page 18 of 36
95. Most of the Banks, from public as well as private sectors, want to build up young work force (18 to 25 years age
group) through massive recruitment drive in the next couple of years.
Which of the following can be a probable cause for the recruitment drive?
(a) Very few existing employees have professional qualifications in finance.
(b) Banks have not hired anyone in the last 10 years resulting in more employers of older generation among the
workforce.
(c) Banks have now decided to hire even persons with 10+2 qualification.
(d) A large number of bank employees are going to retire in the next five years.

96. There has been considerable increase in the number of mobile phone users in India in the past couple of years as
compared to the increase during the previous couple of years.
Which of the following is a probable cause for the above phenomenon?
(A) Cost of mobile phones decreased drastically due to intense competition among the manufacturers of mobile
phones.
(B) The purchasing power of the individuals increased considerably in the past five years.
(C) Mobile phones enable instantaneous dissemination of information, which is growingly becoming a crucial
part of both professional and personal life for the last couple of years.
(a) Only (A) (b) Only (A) and (B)
(c) Only (C) (d) Only (A) and (C)

97. Higher plant load factor imposed by state ABC is also driving the wind energy developers away from the state
to other states where the tariff is on par with ABC.
Which of the following can be a possible cause for higher load factor imposed by the state ABC?
(a) The power generated in the state ABC through tidal power and coal is enough to cater to the needs of the
state.
(b) Wind power developers have to invest more in infrastructure in the state ABC than in any other state.
(c) A lower load factor is not beneficial to the state ABC.
(d) Load factor does not affect the profitability of commercial wind power generation.

98. Vir Sanghvi: “The problem with being a celebrity writer is that all too often the celebrity takes over and the
writer is ignored. He is more read about than read.”
Which of the following statements substantiates the opinion expressed above?
(a) The celebrity writer’s own life is often a case of distraction for the readers.
(b) The celebrity writer’s own life raises more interest than the lives of his fictional creations.
(c) Writers who are not celebrities are widely read than read about.
(d) It is not easy to find readers interested in the works of celebrity writers.

99. The number of road accidents recorded during this year is 300, which is 60% of the total number of accidents
recorded during last year. This trend clearly shows that people have increased awareness about traffic rules.
Which of the following, if true, most seriously weakens the above argument?
(a) The number of drunken-driving accidents this year is less than that of last year.
(b) The number of cases of traffic violations last year is greater than that this year.
(c) The number of unrecorded road accidents this year is 500 while that of last year is 200.
(d) This year, local schools and colleges have included a mandatory ‘no motor vehicle for students’ policy.

Head Office: 127, Zone II, MP Nagar, Bhopal |+91-7676564400| https://www.toprankers.com Page 19 of 36
100. An article in a science magazine: Animal experimentation is necessary in laboratories for conducting research
on development of vaccines and cures for human illnesses. Animals are harmed in the process. But what will
happen to research on AIDS, heart diseases, and cancer if animal experimentation is completely stopped?
Which of the following statements is consistent with the article?
(a) The harm done to animals cannot be clearly measured.
(b) The discovery of anesthesia came from observing humans being insensitive to pain during “ether parties”,
an entertainment involving ether inhalation
(c) There is ample evidence demonstrating the harmful effects that polio vaccine has had on human health
(d) The harm done to humans by not experimenting on animals is far worse than harming animals by
experimenting on them

101. Medical researches found that an extract from roots of suropa, a tree very rare in nature, can be used to prepare
medicine which can cure many diseases. To make one kilogram of medicine, one has to cut down as many as
4,000 trees. Hence, it is concluded that continued manufacture of such medicine would inevitably lead to the
extinction of supora.
Which of the following, if true, weakens the above argument?
(a) The supora is found in deep forests which are generally inaccessible
(b) The cost of acquiring supora roots makes the drugs very expensive
(c) The drugs made from root extract of supora tree need to be dispensed through a central authority.
(d) Supora can be easily propagated and grown under cultivation

102. The unmanned satellites are very important to us, because they are extremely useful in communications and
weather forecast. These satellites can be repaired while in orbit only by astronauts. Hence, we cannot do away
with the space flights that carry astronauts into space.
Which of the following, if true, would most seriously weaken the argument above?
(a) These satellites, once become technically obsolete, pose a grave threat to the new satellites
(b) Computers also play a major role in communications and weather forecast.
(c) The costs involved in repairing satellites is so high that it is more viable financially to send a new satellite
(d) There are many social activities in which the government needs to invest.

103. Metro Rail Authority (MRA) is unable to avoid delays in schedules of trains because it is taking a longer time
for passengers to get off the train at railway stations due to crowding. The number of metro train commuters is
projected to increase by 25% in the next ten years while the MRA is planning to increase the number of train
trips per day by 8% only. MRA is confident that an increase of 8% is sufficient to ensure that there would not be
increase in delays due to crowding.
Which of the following, if true, provides strength to MRA's confidence?
(a) There is not going to be any increase in the other modes of public transport facilities in the next ten years.
(b) A major part of the projected increase in the number of commuters is going to occur during non-peak hours
during which time not many trains are used.
(c) The planned increase of 8% can be achieved by MRA by scheduling the maintenance during non-peak hours.
(d) MRA is also planning to increase the number of buses that connect various places to metro railway stations
by 25%.

104. The Scandinavian Lutherans, who turned away from the excesses of the medieval Catholic Church, were
concerned about equality-especially the disparity between the rich and the poor. They thought that individuals
had some inherent rights that could not just be bestowed by the powerful, and this may have opened them to the
idea of rights for women. Lutheran state churches in Denmark, Sweden, Finland, Norway and Iceland have had
female priests since the middle of the 20th century, and today, the Swedish Lutheran Church even has a female
archbishop.
If the statements above are true, which of the following must be true?
(a) There is not much religion at all in these countries.
Head Office: 127, Zone II, MP Nagar, Bhopal |+91-7676564400| https://www.toprankers.com Page 20 of 36
(b) Men aren’t church goers.
(c) Females are seen as having the same rights as men.
(d) There is no obsessive focus on sins

105. The nuclear deal between Iran and the US is evidence of Obama's much-trumpeted commitment to a nuclear
weapons-free world. And of course, non-proliferation initiatives like this are an essential part of that process.
Which of the following, if true, casts the most serious doubt about the US’s commitment to a nuclear weapons-
free world?
(a) Countries that possess nuclear weapons will take steps to disarm them within the next six months.
(b) Countries without nuclear weapons have decided not to acquire nuclear weapons.
(c) Israel's nuclear arsenal gives it the status as the only country in the Middle East actually with nuclear
weapons.
(d) Some countries that have their own nuclear arsenals have decided not to modernise them.

Directions(Q.106-Q.108): In each question below, some statements are given which are followed by
conclusions (1), (2), (3), and (4). You have to take the given statements to be true, even if they seem to be at
variance with the commonly known facts, and then decide which of the conclusions logically follow(s) from the
given statements, disregarding commonly known facts.

106. Statements:
Some papers are balls.
All cars are balls.
No ball is a gel.
(a) No gel is a car (b) Some gels are paper
(c) All cars are gel (d) No paper is a gel

107. Statements:
Some notes are words.
All words are watches.
All watches are chairs.
(a) No notes is a chair. (b) Some words are not chairs.
(c) No notes is a watch. (d) All words are chairs.

108. Statements:
Some plates are not awards.
All awards are tests.
All tests are rides.
(a) Some awards are rides (b) No awards is a ride.
(c) Some plates are not tests. (d) All plates are tests.

109. If 'broccoli' is related to 'green' then 'tomato' is related to which of the following in the same way ?
(a) Red (b) Purple (c) Black (d) White

110. If the 1st day of a month is on the third day after Monday, then what day will be on the 17 th day of the month?
(a) Saturday (b) Sunday (c) Wednesday (d) Thursday

111. If 'APPROXIMATELY' is coded as 196, 'BAG' is coded as 16 and 'IMAGES' is coded as 49, then what will be
the code for 'DELICATE' ?
(a) 25 (b) 49 (c) 81 (d) 144

Head Office: 127, Zone II, MP Nagar, Bhopal |+91-7676564400| https://www.toprankers.com Page 21 of 36
112. If each alphabet of the English alphabetical order is assigned a number which is a composite number in the
increasing order such that A - 4, B - 6, C - 8 and so on, then what will be the code for the word "HEART"?
(a) 122733206 (b) 10121668 (c) 151042832 (d) 68161820

Directions (Q.113-Q.115): These questions are based on the following information.


Each of five persons Alfa, Beta, Gamma, Delta and Zeta have different efficiencies in completing a work. Each
of them is of different height. All of them are given same amount of work. It is known that Beta is more
efficient but shorter than Alfa. Gamma takes least time to complete the given work and is the shortest. Delta
takes more time than Zeta to complete the work but is shorter than Zeta. Alfa is less efficient than Zeta but
takes less time than Delta to complete the work. Zeta is taller than Beta who is taller than Delta.

113. Who is the least efficient?


(a) Beta (b) Zeta (c) Alpha (d) Delta

114. Who is the second tallest?


(a) Alpha (b) Zeta (c) Beta (d) Cannot be determined

115. If the persons are ranked as per their height and efficiencies, such that the tallest will be given first rank, second
tallest will get second rank and so on and the most efficient will get first rank, second most efficient will get
second rank and so on, it can be observed that no person got the same rank in two categories, then who is the
third tallest and the third efficient respectively?
(a) Zeta, Delta (b) Beta, Zeta (c) Delta, Alpha (d) Zeta, Beta

Directions(Q.116-Q.120): These questions are based on the following information.

A courier service delivered 7 packs – H, J, K, M, T, X, Z – in seven consecutive days of a week to different


places in the city. It delivered some packs in the morning slot and some in the evening slot.

All packs that were delivered in the morning slot were delivered after the packs that were delivered in the
evening slot.
(i) The number of packs that were delivered in the morning slot was one less than that of the packs delivered
in the evening slot.
(ii) K was the first to be delivered.
(iii) Z was delivered before X, but they were delivered in different slots.
(iv) J was the sixth to be delivered.
(v) If Z was third to be delivered then J must be the fifth to be delivered.
(vi) If M is delivered in the morning slot then H must also be delivered in the morning slot.
(vii)H and T were delivered on consecutive days in any order.

116. Which pack was delivered on the seventh day?


(a) M (b) Z (c) X (d) Either (1) or (2)

117. Which of the following is the order of packs which were delivered in the evening slot?
(a) Z, K, M, X (b) K, Z, M, H (c) K, Z, M, T (d) Either (2) or (3)

118. M was delivered on which day?


(a) Seventh (b) Second (c) Sixth (d) Fifth

119. On which day was the pack H delivered?


(a) Third (b) Fourth (c) Fifth (d) Cannot be determined

Head Office: 127, Zone II, MP Nagar, Bhopal |+91-7676564400| https://www.toprankers.com Page 22 of 36
120. Which of the following is/are definitely true?
(a) M was delivered in the evening slot
(b) X was delivered on the sixth day
(c) Z was delivered on the next of J
(d) None of these

Directions (Q.121 to Q.123): These questions are based on the following information.
In a group of six persons – A, B, C, D, E and F – two persons are colleagues, three persons are friends, two
persons are neighbours, two persons are siblings, and four persons are classmates.
(i) D has no friends or siblings. Those who are siblings are not colleagues.
(ii) E and A are classmates, but have no neighbours.
(iii) Neither F nor B is employed and one among the other four is their common friend, who is neither C nor A.
(iv) Those who are friends are not siblings. C has a neighbour and has a sibling.
(v) Two of the classmates are neighbours, who are also siblings.
(vi) E is unemployed and F’s father has only one child.

121. Who are siblings?


(a) B and C (b) F and E (c) E and B (d) C and F

122. Who are colleagues?


(a) AC (b) AD (c) DC (d) Cannot be determined

123. If A has no colleagues, then who among the following is C’s colleague?
(a) D (b) E (c) A (d) B

Directions(Q.124 and Q.125): These questions are based on the following information.
Seven persons–P, Q, R, S, T, U and V–are in a queue for Bus tickets. R and U do not like each other, hence they
stand as far away as possible. S and P are friends, so they stand one after the other respectively at the front. T
stands in the middle of the queue.

124. Which of the following pairs cannot be immediately in front of or immediately behind T?
(a) R and Q (b) U and V (c) P and R (d) V and R

125. Who among the following cannot be either at the front or at the rear end of the queue?
(a) P (b) S (c) U (d) Q

126. K is the daughter in law of A. C is the mother of E. V is the son of L. E is the son of J. C is the sister of B who
is married to A. L is the husband of K.
How is K related to E?
(a) Aunt (b) Sister (c) Sister in law (d) Cousin

127. Four number pairs are given out of which three are alike in some manner and one is different.
Select the option in which the number pair is different.
(a) 7 : 350 (b) 2 : 10 (c) 11 : 132 (d) 5 : 130

128. Find the missing letter in the series.


F, H, K, O, T, Z, G, ___
(a) O (b) I (c) R (d) J

Head Office: 127, Zone II, MP Nagar, Bhopal |+91-7676564400| https://www.toprankers.com Page 23 of 36
129. Select the option in which the two group of letters are related in the same way as the two group of letters given
below.
ABGJT : FXJHU :: ?
(a) NKWQI : SPSNJ (b) GHMPV : LDONW
(c) DURCK : IQUAL (d) EFLOU : IBNQV

130. Select the word pair in which the two words are related in the same way as are the two words in the following
word pair (in the same order).
Travel : Itinerary :: ?
(a) Work : Calendar (b) School : Timetable
(c) Trains : Tickets (d) Town : Maps

131. Legal Principles:


1. Everybody is under a legal obligation to take reasonable care to avoid act or omission which he can foresee
would injure his neighbour.
2. The neighbour for this purpose is any person whom he should have in his mind as likely to be affected by
his act.
3. A person is liable for the tort of negligence if he breaches a legal duty of care he owes to the plaintiff, and
the plaintiff suffered a damage resulting from this breach.
4. The damage so caused must be an immediate cause of the act of negligence and not a remote cause.
Factual Situation: Mr. R S Gupta is a 25 year old person who likes traveling a lot and it is his dream that he can
travel all the major tourist destinations in India. But due to difficult financial situation at home, he does not have
sufficient money to travel to all those places. one of his friend gave him the idea to becoming a travel blogger as
with this method he will be able to travel and earn money through his blogging simultaneously. One day he got
held up in some work and arrived late at the station. When he arrived his train was already leaving so he rushed
to board the moving train and in doing so pushed Mr. S who was walking along with a heavy package, containing
fire crackers. As a result, the package slipped from his hand and the crackers exploded injuring a boy standing
closely. A suit was filed against Mr. R S Gupta, by the boy, claiming damages. So, now decide which of the
following statements can most plausibly be inferred from the application of the given principles to the given
facts.
(a) Mr. R S Gupta is not liable, because he did not know anything about the contents of the package.
(b) Mr. R S Gupta is not liable, because Mr. S should not have carried such a package in a crowded place like
Railway station.
(c) Mr. R S Gupta is liable, because he was under an obligation not to push Mr. S.
(d) No one was liable as it was a misfortunate accident.

132. Legal Principles:


1. An occupier is liable to a trespasser in respect of some wilful act intended to cause harm or done with
reckless disregard.
2. It is a well-known principle that if a person enters upon another’s land and stays on it, the act is connoted
as continuing trespass.
3. Under the strict liability rule, the law makes people pay compensation for damages even if they are not at
fault.
4. An occupier or owner of land owes a duty to warn a suspected trespasser of deadly conditions on the land
which would be hidden to a trespasser, but of which the property owner is aware. If any extra force is used
the liability is transferred.
Factual Situation: Mr. S C Chand is a 40-year-old businessman who has his hands full with different kind of
businesses. Recently he has purchased a new warehouse in the outskirts of New Delhi. He is very over cautious
about certain situation and many of his co-workers think that it is a not a good habit in business. And due to
this nature, he got the fence electrified on account of his fear of dacoity of the goods left in the warehouse. To
Head Office: 127, Zone II, MP Nagar, Bhopal |+91-7676564400| https://www.toprankers.com Page 24 of 36
make the electrified fencing legal, there was a clear warning about the electrified fencing placed outside the
warehouse. There was a playground nearby wherein the children had been playing over a period of time. One
day, the cricket ball of the children went into the fence and a child running after the ball touched the fence
inadvertently. The child suffered from the shock. The parents of the child field a suit against Mr. Chand. So,
now decide which of the following statements can most reasonably be inferred from the application of the
given principles to the given facts.
(a) S C Chand will not be liable, because the child was a trespasser.
(b) S C Chand will not be liable, because his fencing was legitimate.
(c) S C Chand will be liable, because he must have taken note of the adjacent playground.
(d) S C Chand will not be liable since children negligent act is a matter which is not in his hands.

133. Legal Principles:


1. Every person has a right to complete immunity of his person from physical interference of others, except in
so far as the contact may be necessary under the general doctrine of privilege.
2. Nothing which is not intended to cause death, is an offence by reason of any harm which it may cause, or be
intended by the doer to cause, or be known by the doer to be likely to cause, to any person for whose benefit
it is done in good faith, and who has given a consent, whether express or implied, to suffer that harm, or to
take the risk of that harm.
3. So long as a doctor follows a practice acceptable to the medical profession of that day, he cannot be held
liable for negligence merely because a better alternative course or method of treatment was also available or
simply because a more skilled doctor would not have chosen to follow or resort to that practice or procedure
which the accused followed.
4. A professional may be held liable for negligence on one of the two findings: either he was not possessed of
the requisite skill which he professed to have possessed, or, he did not exercise, with reasonable competence
in the given case, the skill which he did possess.
Factual Situation: Mr. O P Sen is a famous businessman who is of 55 years of age. One day, he collapsed in
his office in the middle of a meeting and when he was taken to the hospital, he was diagnosed with cancer and
it has reached the terminal stage. It was clear to everyone including the personal physician of the patient, Dr. J
P, that the patient did not have much time. At that stage, the specialist doctor in charge of the treatment, Dr. J L,
administered a drug which was at the stage of experimentation without the consent of the patient. The experiment
had established that the drug could control the spread of cancer cells to some extent. Mr. O P Sen died soon after.
When the relatives of Mr. Sen came to know about this incident, they filed a suit against the hospital and Dr. J
L, charging them for assault and battery, i.e. unjustified physical interference. So, now decide which of the
following statements can most probably be inferred from the application of the given principles to the given
facts.
(a) Dr. J L is liable, since he has acted without the consent of the patient.
(b) Dr. J L is not liable, since he was motivated by the welfare of the patient.
(c) Dr. J L is not liable, since it was in the general public interest that the new drugs should be developed.
(d) Dr. J L is liable since he should have notified the patient’s relatives instead who would have been able to
make a more informed decision.

134. Legal Principles:


1. Both assault and battery are the types of intentional tort.
2. A person, intentionally causing harm to others, is liable for battery.
3. A person is said to have intended the harm, in so far as he brings about the harm purposefully or
knowingly.
4. Battery is only considered when there is an actual physical contact without the consent of the person to
harm the person.
Factual Situation: Mr. S K Gupta is 35-year-old person who is a teacher by profession and teaches in a
coaching centre called Bright Future Academy. Although Mr. S K Gupta follow a healthy lifestyle and do
proper exercise to keep his body fit. But from past couple of days, he is suffering from back pain. One day
Head Office: 127, Zone II, MP Nagar, Bhopal |+91-7676564400| https://www.toprankers.com Page 25 of 36
when Mr. Gupta was moving around the class room in the course of teaching, he suddenly felt intense pain in
the back so he decided to sit down for some time. As he was moving backwards to reach his chair, a student,
X, who did not know the fact that Mr. Gupta had back issues, saw a scorpion moving underneath the chair and
immediately rushed to attack the scorpion, and in that process, moved the chair. Meanwhile, Mr. Gupta landed
straight on the ground, rather in swift motion, and suffered a slipped disc. A suit was filed against the student
for battery. So, now decide which of the following statements can most plausibly be inferred from the
application of the given principles to the given facts.
(a) The student is not liable, since he wanted to save the teacher and others from the scorpion.
(b) The student is liable, since he knew that the teacher would be taking her seat.
(c) The student should be held additionally liable for the lack of attention to the teaching.
(d) The student could have given a vocal warning about the scorpion.

135. Legal Principles:


1. Both assault and battery are the types of intentional tort.
2. Battery is only considered when there is an actual physical contact without the consent of the person to harm
the person.
3. Assault is a planned attempt to violently harm another person.
4. No physical contact required to constitute the offence of assault.
Factual Situation: Miss P is a 26 year old student who is pursuing her Ph.D in Law in the National X Y Z
collage, Mohali, Chandigarh. As the college does not provide hostel for Ph.D students, she rented a flat few
kilometers away from the college. Mr. C is a 26 year old person who is also pursuing his Ph.D in Law in the
same college. And when he saw Miss P in the college, he started following her, making silent telephone calls
and writing on her door, which could be described as stalking, he had written over 800 letters to her in the space
of 4 months, which caused her to suffer panic attacks. One day Mr. C had delivered a letter by hand and when
Miss P read the letter, it caused her to believe that Mr. C had completely lost his mind and would use force
against her. Further, there was medical evidence that Miss P was suffering from a clinical state of depression
and anxiety. So, now decide which of the following statements can most plausibly be inferred from the
application of the given principles to the given facts.
(a) This was not a case of assault since there was no perceivable or imminent harm.
(b) Mr. C’s words were not backed by any physical actions, which is requisite for assault.
(c) Mr. C’s actions were enough to cause a reasonable apprehension in Miss P’s mind; thus, he is liable for
assault.
(d) Mr. C’s mental health is not stable so he cannot be punished.

136. Legal Principles:


1. Assault is the attempt to commit battery.
2. Threat of violence is enough for assault. No physical contact is necessary.
3. Create reasonable apprehension in the plaintiff’s mind that immediate force will also be used.
4. The victim’s fear must be a direct response to a threat that is imminent.
Factual Situation: John is 16-year old boy who lives in central street, New Delhi. He is the only child and he is
a little hot-headed, well-built, plays passionately and fights, both verbal and physical, even more passionately.
So all the other students in his school and his other friends are a little intimidate by him. There is play ground
near his house and he often used to go there to play cricket. One day when he was playing cricket, the ball rolled
onto an old man’s lawns, which was enclosed by a fence. The gate was locked. The old man refused to let him
and other children in. The boy roared, “if it were not for your gray hairs, I would tear your heart out!”. The old
man runs hobbles in as quickly as his arthritic knees can take him and calls the police, the living daylights almost
sucked right out of him. Can the court accept a claim of assault? So, decide which of the following statements
can most plausibly be inferred from the application of the given principles to the given facts:
(a) No, john was at a great distance from the man, who was also protected by the fence.
(b) Yes, since elderly folks can be easily spooked and in any case have several ailments which will compound
even the smallest health issue.
Head Office: 127, Zone II, MP Nagar, Bhopal |+91-7676564400| https://www.toprankers.com Page 26 of 36
(c) Yes, as john looked capable enough to cause an impression that he could carry out his threat.
(d) No, he did not create a reasonable apprehension of injury.

137. Legal Principles:


1. Nothing is an offence which is done by any person who is justified by law, or who by reason of a mistake of
fact and not by reason of a mistake of law in good faith, believes himself to be justified by law, in doing it.
2. Nothing is an offence which is done by a person who is, or who by reason of a mistake of fact and not by
reason of a mistake of law in good faith believes himself to be, bound by law to do it.
3. False imprisonment is the arrest of the individual by the police officer or private person without lawful
authority.
4. The defense to false imprisonment includes consent of the plaintiff or voluntary assumption of the risk,
probable cause and contributory negligence.
Factual Situation: Mr. H Sharma is a 25 year old person who was rushing to the hospital as one of his closest
friends got into a terrible accident where his car flipped over. When he reached the hospital, one of the doctors
quickly gave him a prescription and asked him to buy some medicines from a nearby pharmacy. So, he went to
his local pharmacy to get some medicines. When the pharmacist, Mr. M Singh, take a look at the prescription
and medicine mentioned there, he doubted the authenticity of the prescription. So, he stalled for time and
instructed Mr. H Sharma to wait, while simultaneously and without Mr. H Sharma’s knowledge called the police.
When the police arrived, they arrested Mr. H Sharma. The later police verified with his doctor that the
prescription was authentic and that it was meant for him. After this incident, the patient sued the pharmacy and
its employees. He received Rs. 20,000 in damages. Mr. M Singh appealed the decision. So, decide which of the
following statements can most plausibly be inferred from the application of the given principles to the given
facts:
(a) Mr. M Singh would get relief since he had a reasonable suspicion and acted accordingly. He did not use
force or physical restraint.
(b) The appeal would fail since Mr. M Singh acts were not reasonable.
(c) The appeal would fail since Mr. H Sharma had to go through such mental agony.
(d) The appeal would succeed since Mr. M Singh’s acts were well within his rights as a pharmacist.

138. Legal Principles:


1. The owner of immovable property is entitled to the column of airspace above the surface,
2. However, the owner’s right to air and space above his land is restricted to such height as is necessary for the
ordinary use and enjoyment of his land and the structures on it.
3. Water flowing below your land is not yours though you can use it.
4. All mineral resources below the land belongs to the State.
Factual Situation: X Y X Pvt. Ltd. is a world renowned company which has several branches in various
countries. Its main branch is located in the capital city of India, New Delhi. The main business of the company
is related to GPS as they provide navigation and GPS services and devices to the customers. Recently various
other companies started to venture into this area of business and X Y X Pvt. Ltd. is facing a great competition in
the market due to this. So, the company came up with a unique method and decided to update their overall
navigation system. This unique method involved taking aerial photographs for mapping purposes. The company
employed several employees to this particular task. One day when Mr. John with his crew was taking aerial
photographs for mapping purposes on a particular area. They were spotted by Sam, when the helicopter was
passing several hundred feet over their house and taking photographs. He sued X Y X Pvt. Ltd., alleging trespass
of his airspace. So, decide which of the following statements can most plausibly be inferred from the application
of the given principles to the given facts:
(a) Clicking photos of his house constitutes a trespass, hence X Y X Pvt. Ltd. is liable.
(b) There was no trespass since X Y X Pvt. Ltd. did not go on to his land or property.
(c) Sam does own the entire airspace above his sky, but only to the reasonable extent that is necessary for his
enjoyment. Hence, X Y X Pvt. Ltd. is not liable for trespass.

Head Office: 127, Zone II, MP Nagar, Bhopal |+91-7676564400| https://www.toprankers.com Page 27 of 36
(d) The fact that the helicopter was right over his house and clicking pictures of it, both of which bothered Sam,
is enough to sustain a claim of trespass due to interference on his enjoyment.

139. Legal Principles:


1. All citizens shall have the right to freedom of speech and expression and to form associations or unions;
2. All citizens shall have the right to move freely throughout the territory of India and to reside and settle in
any part of the territory of India; and
3. All citizens shall have the Fundamental Right to carry on any occupation, trade or business.
4. Reasonable restrictions on the exercise of such rights can be imposed by law in the interest of the general
public.
Factual Situation: Mr. P Singh is a 24 year old person who used to live in near the slum areas of Mumbai. A
large number of persons had been carrying on the business of dyeing and printing in slum area of Mumbai for
the last 25 years. This business of dyeing and printing provides employment to about 30000 families. From these
business places untreated dirty water was being discharged on the roads thereby causing damage to the public
health. A notice, therefore, was given to close this business till necessary measures to protect public health as
provided under the environmental statutes were taken by those businessmen. When Mr. P Singh, who just
completed his BALLB degree from a reputed law school in India and returned home to find a good job in legal
field, got to know about this situation, he gathered everyone and told them that the notice violated their
fundamental right to carry on any occupation. After that they decided to approach the court. So, decide which of
the following statements can most plausibly be inferred from the application of the given principles to the given
facts:
(a) Notice cannot be justified as it will cause loss of employment to 30000 families
(b) Notice cannot be justified as it amounts to violation of the Fundamental Right of the persons who have been
carrying on the business for the last 25 years
(c) The notice cannot be justified on the ground of damage to public health as the persons in that area have been
voluntarily residing for long and have become used to that environment
(d) The notice can be justified as the right to business is not absolute and reasonable restriction can be imposed
by law in the interest of the public

140. Legal Principles:


1. A contract is a promise or set of promises for the breach of which the law gives a remedy.
2. Any words or conduct by one or both parties that communicate a legally enforceable promise will constitute
a contract.
3. Mere promise without a proper follow-up does not create a binding legal obligation.
4. Acceptance must be communicated to the offeror himself. A communication to any other person is no
communication in the eyes of law.
Factual Situation: Mr. R K Sharma is a 50 year old businessman who has a small scale business in the city of
Mumbai. He recently set up a new office for his business in Kanpur and he was planning to move their in next
few months. So he wants to purchase Mr. V K Singh’s residential house situated in Indira Vikas Colony, Kanpur
for Rs. 5 lakh. He writes to Mr. V K Singh that he would assume his offer has been accepted unless he hears to
the contrary from him (Mr. V K Singh) within a month. Mr. V K Singh does not reply but directs his agents Miss
F, a property dealer not to sell the above said property to anybody else because he (Mr. V K Singh) wants to sell
it to Mr. R K Sharma. However Miss F sell the property to Miss K. Can Mr. R K Sharma file a suit seeking
enforcement of contract against Mr. V K Singh. So, decide which of the following statements can most plausibly
be inferred from the application of the given principles to the given facts:
(a) No, he cannot because an offer cannot consider to be accepted unless its acceptance should be communicated.
(b) Yes, Mr. R K Sharma can file a suit against Mr. V K Singh.
(c) No, because Miss K bought the property and now he is the owner.
(d) No, Mr. R K Sharma cannot file a suit against Mr. V K Singh.

Head Office: 127, Zone II, MP Nagar, Bhopal |+91-7676564400| https://www.toprankers.com Page 28 of 36
141. Legal Principles:
1. Nothing is an offence which is done by accident or misfortune, and without any criminal intention or
knowledge in the doing of a lawful act in a lawful manner by lawful means and with proper care and caution.
2. Nothing is an offence which is done in the exercise of the right of private defence.
3. The right of private defence of the body extends to the voluntary causing of death or of any other harm to
the assailant, if the offence reasonably causes the apprehension that death, or grievous hurt will otherwise be
the consequence of such assault.
4. If in the exercise of the right of private defence against an assault which reasonably causes the apprehension
of death, the defender be so situated that he cannot effectually exercise that right without risk of harm to an
innocent person, his right of private defence extends to the running of that risk.
Factual Situation: Mr. F is a 50 year old retired army soldier who is living with his wife and two sons in the
newly build home near Mumbai city. He recently finished the construction of the house after buying the plot of
land few months back. Now when they were moving to the new house, his two son Mr. X and Mr. Y had a huge
fight regarding the allocation of separate rooms for them as both of them wanted the same room. Eventually Mr.
F decided to teach them a lesson and provided them a single room where both have to live with each other. Later
X was attacked when he was asleep at night by his brother Y who tried to strangulate him. Apprehending
imminent death the accused aimed a blow at his assailant brother with a piece of bamboo on which he could lay
hand and the blow struck the head of his intervening father ‘F’ as a result of which he ultimately died. So, decide
which of the following statements can most plausibly be inferred from the application of the given principles to
the given facts:
(a) Father ‘F’ was killed by ‘X’ intentionally during the fight.
(b) Father ‘F’ was killed by X and Y accidentally during the fight.
(c) ‘X’ is liable as he should have been more careful.
(d) Accused ‘X’ is not liable and the blow fell on the head of his father by accident and misfortune.

142. Legal Principles:


1. Generally an agreement without consideration is not valid. Therefore, in order to make a valid agreement,
some consideration which may have some value in the eyes of law, is essentially required.
2. A contract is an agreement between two or more parties to perform or to refrain from some act now or in the
future.
3. An Agreement is a promise between two entities creating mutual obligations by law.
4. If a party to a contract agrees to it under the undue influence of any other party then the party under the
undue influence may refuse to perform in accordance with the agreement.
Factual Situation: Mr. W is a 50 year old person who is doing great in his career as he recently got promoted
to the post of general. He was very happy with his promotion and decided to buy a new BMW luxury car. After
he bought the new car, he stopped using his old car which a classic old style car. As he has an old car of which
he makes seldom use. He voluntarily enters into an agreement with Mr. S to sell this car for rupees ten thousand.
Thereafter one Mr. X approaches Mr. W and offers to buy that car for rupees 1 lakh as the car was one which
Mr. X has been searching for long. Now, Mr. W wants to cancel his agreement with Mr. S and refuses to deliver
the car to him saying that consideration price for the car promised by Mr. S is negligible and therefore, agreement
with him cannot be said to be valid one. So, decide which of the following statements can most plausibly be
inferred from the application of the given principles to the given facts:
(a) Mr. W can cancel his agreement with Mr. S as the consideration involved in that is really inadequate
(b) Mr. W cannot cancel his agreement with Mr. S as the sale of car for rupees ten thousand was voluntary and
this price has some value in the eyes of law
(c) Mr. W can cancel his agreement with Mr. S as he was ignorant about the value/price of the car for which it
could be sold
(d) Mr. W can cancel his agreement with Mr. S as he is entitled to get full market value/ price of his car

Head Office: 127, Zone II, MP Nagar, Bhopal |+91-7676564400| https://www.toprankers.com Page 29 of 36
143. Legal Principles:
1. The state shall not discriminate, either directly or indirectly, on the grounds of sex, race, religion, caste,
creed, sexual orientation, marital status, disability, pregnancy, place of birth, gender orientation or any other
status.
2. Direct discrimination occurs when for a reason related to one or more prohibited grounds a person or group
of persons is treated less favourably than another person or another group of persons in a comparable
situation.
3. Indirect discrimination occurs when a provision, criterion or practice which is neutral on the face of it would
have the effect of putting persons having a status or a characteristic associated with one or more prohibited
grounds at a particular disadvantage compared with other persons.
4. Discrimination shall be justified when such discrimination is absolutely necessary in order to promote the
well-being of disadvantaged groups, such as women, Dalits, religious minorities, sexual minorities or
disabled persons.
Factual Situation: On 8th March, 2011, on the occasion of International women’s day, every state decided to do
something for the women’s in their states as it was declared by the National women commission to do something
for the women of the states and of the nation on 8 th March. So, each and every state send their resolution or
proposal to the departments in their states about the initiatives for women. As like this the Governor of the state
of MP also made the proposal without sending it to department. Hence, for the occasion of the same Governor
of MP ordered the release of all women prisoners who were serving a sentence of less than one- year
imprisonment to mark the occasion of International Women’s Day. Presumed that the Governor of MP, also
made another order needed that the release of all persons under the age of 30 and over the age of 60 who were
serving a punishment or sentence of less than one year imprisonment. So later, it was claimed that under these
rules, the order is discriminatory. Now decide whether the order is discriminatory or not:
(a) The order is Directly Discriminatory.
(b) The order is Indirectly discriminatory.
(c) The order is Not discriminatory.
(d) The order is Discriminatory, but justifiable.

144. Legal Principles:


1. Section 18 of the Trade Unions act deals with immunity from civil suit in certain cases.
2. No suit or other legal proceeding shall be maintainable in any Civil Court against any registered Trade Union
or any office-bearer; or member thereof in respect of any act done in contemplation or furtherance of a trade
dispute to which a member of the Trade Union is a party on the ground only that such act induces some other
person to break a contract of employment.
3. Or that it is in interference with the trade, business or employment of some other person or with the right of
some other person to dispose of his capital or of his labour as he wills.
4. A registered Trade Union shall not be liable in any suit or other legal proceeding in any Civil Court in respect
of any tortious act done in contemplation or furtherance of a trade dispute by an agent of the Trade Union if
it is proved that such person acted without the knowledge of, or contrary to express instructions given by,
the executive of the Trade Union.
Factual Situation: Toloman, the secretary of a registered Trade Union was living his best life with the family.
He was good at his work also as in he was committed and profession to the work. One day, his daughter asked
him that she wanted to go for higher studies to another country. By this he thought that for higher studies of the
daughter maybe he need a loan so he found the bank for the loan. One day, Toloman took a loan from a bank for
the higher education of his daughter. During the course daughter found a boy and she started dating with him
and soon after completing the course she was married to the same boy who was an NRI Engineer. After knowing
all this and by the years of the daughter’s higher studies Toloman did not repay the loan and he never told why
he is not repaying the loan to the bank. The bank demanded the payments from Toloman and warned him that
the Bank will take suitable legal action against him. Now recognise the legal position in this regard.

Head Office: 127, Zone II, MP Nagar, Bhopal |+91-7676564400| https://www.toprankers.com Page 30 of 36
(a) The bank can file a suit for recovery of the loan amount against Toloman as he took the loan for a personal
purpose and in such case, no immunity will work.
(b) The bank cannot initiate any action against Toloman as he is the Secretary of a Registered Trade Union.
(c) The bank can recover the loan amount from the Trade Union as Toloman is the secretary of the union.
(d) As Toloman did not use the loan amount for his own use, the loan amount was for his daughter’s use and
hence, no action can be initiated against him.

145. Legal Principles:


1. On the death of the husband, the widow shall inherit the property of her deceased husband along with children
equally.
2. A widow is a limited heir, acquires the property for her life but she is the owner of the property thus inherited
as a tenant.
3. A widow cannot claim the property of the deceased on the date when the question of succession opens, she
has remarried.
4. A female acquiring property in any way has the absolute title to the property.
Factual Situation: Ramsiya, one of the sarpanch of the Raigarh village was living his life with the best resources.
He has two brothers also who stays with Ramsiya only. Ramsiya has farms and some family property also which
is properly divided between him and his brothers. One evening Ramsiya was not feeling good so he told his
children to stay at home only and he told the shares of him in the family property to the children but on the same
day, Ramsiya died due to heart attack.
When Ramsiya died, he had 1/3rd share of the family property, which the three brothers Rupesh, Deepak, and
Shyam Kishore inherited from their father, Bhanu. After some years due to some disease Rupesh also died on
September 27 2007 without having any issue. The widow of Rupesh, Ms. Vinita remarried on January 2, 2008.
Then Vinita, asked about the share in the property Deepak and Shyam Kishore refused ‘Vinita’ the share from
Rupesh’s portion and Vinita claimed the entire property belonging to Rupesh on January 31, 2008. Now decide
whether Vinita is eligible for the share or not. Decide.
(a) Vinita cannot inherit the property of Rupesh as the Vinita does not belong to the family.
(b) Vinita cannot inherit the property of Rupesh as Vinita was remarried.
(c) Vinita can inherit the property of Rupesh as her claim was on the date of Rupesh’s death.
(d) Vinita can inherit the property of Rupesh since Vinita does not belong to the family.

146. Legal Principles:


1. Environmental pollution is unwarranted disposal of mass or energy into earth’s natural resource pool such
as water, land, or air that results in long- or short-term detriment to the atmosphere and its ecological health
to negatively impact the living beings and their life both quantitatively and qualitatively
2. Environmental pollution reflects a measurement of contamination of the living and non- living (physical and
biological) constituents of the earth, in a manner and to an extent, to adversely affect the normal optimum
environmental processes
3. Whosoever by his act or omission causes environmental pollution shall be held liable for any loss caused by
such pollution.
4. It shall be no defence in such cases that all due diligence or reasonable care was taken while carrying out the
act or omission in question.
Factual Situation: Hardeep is carrying on a chemical and fertilizer industry near a bank of a river. Hardeep
almost take every precaution for the industry and for the welfare of the society. But when there was survey by
the authorities, they asked the industry that what are they doing for the pollution things, as they are implementing
so many things for other factors, but not for preventing the pollution. So, Hardeep asked his members that what
can be implemented for this, the members suggested different options and after full approval they came on some
kind of implementation. In order to prevent and control any kind of harm to the environment, suitable waste
treatment and disposal plants were installed in the factory. One day, due to some sudden mechanical/technical
problem, these plants ceased to work properly and, therefore, caused environment pollution, which ultimately

Head Office: 127, Zone II, MP Nagar, Bhopal |+91-7676564400| https://www.toprankers.com Page 31 of 36
caused substantial harm to the environment and to the people living around the factory. The chemical and
fertilizer tied to take precautions but they cannot limit the pollution. Victims of such pollution, files a suit for a
suitable remedy. Decide.
(a) Victims cannot succeed as necessary precautions to prevent any harm were taken by Hardeep.
(b) Victims cannot succeed as the mechanical or technical problem was sudden and, therefore, beyond the
control of Hardeep.
(c) Victims can succeed as it is the duty of Hardeep to see that no harm is caused to the environment/people due
to his activity under any circumstances.
(d) Victims could succeed, if the treatment or disposal plant were not installed in the factory.

147. Legal Principles:


1. Under the Hindu Adoptions and Maintenance Act, 1956, no person shall be capable of being taken in
adoption unless he or she is a Hindu.
2. He or she not already been adopted, he or she has not been married, unless there is a custom or usage
applicable to the parties which permit persons who are married being taken in adoption.
3. He or she has not completed the age of fifteen years being taken in adoption.
4. When a Hindu male or a Female want to adopt a son, they should not have a son living whether legitimate
or illegitimate, at the time of adoption.
5. When a male or a Female Hindu want to adopt a daughter, they should not have a daughter or son’s daughter
living at the time of adoption.
Factual Situation: Radhe, a government servant, working in this field for around 10 years. He met the girl Suchi
and they both decided to get married. After few years of age Suchi gave a birth to a boy and Radhe and Suchi
gave the boy named, Tom. When Tom was of 10 years his father Radhe has a thought of giving him to adoption
to his best friend Shyam, as Shyam needs a child. Hence, Radhe being a natural father of Tom, he had given
Tom, a boy aged 10 years, in adoption to Shyam in March 2010 in accordance with the Hindu Adoptions and
Maintenance Act, 1956. In May 2012, Sachin another friend of Shyam liked Tom and he asked to Shyam that
will he give Tom in adoption or not and Shyam agreed on this and gave Tom in adoption to Sachin. Subsequently,
in December 2013, Sachin went to the party where he met one of the colleague from his office Radhe and Radhe
asked Sachin for adopting Tom and Sachin agreed on the same. On the next day, Sachin gave Tom in adoption
to Radhe.
(a) Adoption of Tom by Sachin is valid. (b) Adoption of Tom by Radhe is valid.
(c) Adoption of Tom by Shyam is valid. (d) None of the adoptions is valid.

148. Legal Principles:


1. Whoever intending to take any movable property out of the possession of any person without that person’s
consent, moves that property out of his or her possession, is said to commit theft.
2. Whoever commits theft shall be punished with imprisonment of either description for a term which may
extend to three years or with fine or with both.
3. A thing so long as it is attached to the earth, not being movable property, is not the subject of theft; but it
becomes capable of being the subject of theft as soon as it is severed from the earth.
4. A person who, without lawful excuse, damages any property belonging to another intending to damage any
such property shall be guilty of causing criminal damage.
Factual Situation: Lata, an old lady of 80 years, a citizen of XY city. She used to live with her grand-son Jayesh.
Lata was ill and bed ridden for several months. In those months, she could not tolerate any noise which came
from her locality and it became quite difficult for her to live in peace in her room. One day due to her health
issues she died. After she died, Jayesh hired a cleaner, Diljeet, to clean Lata’s room and throw away any rubbish
things that may be there.
There was a bundle of old newspapers and magazines which Lata had stacked in a corner of her room. Diljeet
asked Jayesh if he should clear away the bundle of old newspapers and magazine, to which he said yes. Diljeet
took the bundle to a municipality dump. While Diljeet was sorting and throwing away the newspapers and
magazines, he was very surprised to find a wall painting in between two magazines. He thought that Jayesh
Head Office: 127, Zone II, MP Nagar, Bhopal |+91-7676564400| https://www.toprankers.com Page 32 of 36
probably would not want this old painting back, especially because it was torn in several places and the colour
was fading. Diljeet took the wall painting home, organized it on a wooden frame and hung it on the wall of his
room. Unknown to him, the painting was an old gem, and worth Rs. 30000. Before organizing the painting
Diljeet pasted it other sheets of paper on it so that it does not tear any more.
Diljeet’s neighbour Radhe discovered that painting belonged to Jayesh. With the motive of returning the painting
to Jayesh, Radhe climbed through an open window into Diljeet’s room when he was away one afternoon and
removed the painting from his house. Decide, Is Radhe guilty of theft or criminal damage?
(a) No, Radhe is not guilty of theft since he took the painting only with the motive of returning to Jayesh.
(b) Yes, Radhe is guilty of criminal damage as he took the painting without Diljeet’s consent.
(c) No, Radhe is not guilty of theft since the person he took the painting from (Diljeet) was not its lawful owner.
(d) Yes, Radhe is guilty of theft as he took the painting out of Diljeet’s possession without his consent.

149. Legal Principles:


1. Malicious Prosecution is defined as a judicial proceeding against the another, from wrongful or improper
motive, without any reasonable and probable cause to justify it.
2. In order to establish malicious prosecution, the prosecution should be terminated in favour of the plaintiff, if
it is so capable to be terminated, or the plaintiff is exonerated of any charge.
3. A person is liable if there must be an absence of an incredible and probable cause. An incredible and probable
cause is an honest belief in the guilt of the accused founded on reasonable grounds.
4. Malicious intention will be said to exist if the prosecution is instituted with a wish to injure the party rather
than to vindicate law and justice.
Factual Situation: Nisha, an employee of JJ Company, was unhappy with certain things and with some people
of the company. She was a regular employee of the company. But because of some reasons Nisha was very upset
with her boss Umesh due to his high-handed attitude and rude behaviour.
Umesh at various occasions in the past had rejected Nisha’s leave application on very flimsy grounds. Once
Nisha told her colleagues that she would teach Umesh a lesson of his life. One day, Umesh called Nisha to his
cabin and told her that she needs to perform as per his expectations otherwise she would not be promoted. After
seeing this inappropriate behaviour of Umesh, she decided to take an action against him.
Nisha filed an FIR against Umesh for sexual harassment. The police authorities started an investigation and
questioned Umesh in his office premises causing immense embarrassment and mental agony to him. Umesh was
also taken in remand by the police authorities for interrogation but at last the charges were dropped against him,
because of the lack of any evidence. Umesh filed a suit complaining malicious prosecution against Nisha and
also for recovering damages. Whether there was malicious intent on the part of Nisha?
(a) No, as she used legal measures to redress her grievance.
(b) Yes, as she intended to get her leave applications sanctioned.
(c) Yes, as Nisha instituted the proceedings with the intent to injure the reputation of Umesh and teach him a
lesson.
(d) No, as Nisha complained about the harassment based on the statement as understood by her.

150. Legal Principles:


1. According to the doctrine of vicarious liability, a person shall be liable for the damage caused by his
employee in the course of employment.
2. An employer shall be liable to the injuries caused to his employee by the negligence of a fellow employee
in the course of employment.
3. A person is an employee of another if the form and the way in which he or she convey out his work is subject
to association and administration of the latter.
4. An employer is required to provide compensation to his or her employees for any injury caused by an
accident arising in the course of employment. The objective of ‘in the course of employment’ mean in the
course of the work which the employee is contracted or obligated to do and which is subsidiary to it.

Head Office: 127, Zone II, MP Nagar, Bhopal |+91-7676564400| https://www.toprankers.com Page 33 of 36
Factual Situation: Messrs. Shyam Chaudhary and co., a Mumbai based company manufactures organic drinks
with the help of persons known as ‘Production worker’. This company covers every area of Mumbai in respect
to the production and supply of drinks.
The production workers are supplied raw materials by the company and are required to used them for making
the drinks and bring the stock of drinks back to the company. The production workers are free to make the juice
with the raw material either in the factory or anywhere else they prefer. Production workers are not restrained to
attend the factory for any fixed hours of work or for any fixed number of days. Neither are they required to make
any fixed number of drinks. The Company verifies whether the drinks adhere to the specified instructions or not
and pays the production workers on the basis of the number of drinks that are found to be of right quality.
Raj Singh is one of the production workers of the Company. He works on daily basis for the company. He was
hit by a truck just outside the premises of the factory while he was heading to home in a nearby metro station.
Raj Singh has applied for compensation from the company. Whether the company is liable to pay compensation
or not?
(a) The company is liable to pay the compensation as Raj Singh is a contracted production worker with the
company.
(b) Since the injury did not arise in the course of employment, the company would not be liable to pay the
compensation even though Raj Singh is an employee of the company.
(c) The company is not liable to pay compensation as the injury to Raj Singh was not caused by an accident
arising in the course of employment.
(d) The company is liable to pay the compensation.

Head Office: 127, Zone II, MP Nagar, Bhopal |+91-7676564400| https://www.toprankers.com Page 34 of 36
USE FOR ROUGH WORK

Head Office: 127, Zone II, MP Nagar, Bhopal |+91-7676564400| https://www.toprankers.com Page 35 of 36
USE FOR ROUGH WORK

Head Office: 127, Zone II, MP Nagar, Bhopal |+91-7676564400| https://www.toprankers.com Page 36 of 36

You might also like